Đến nội dung

Hình ảnh

Chứng minh rằng: $\left | \frac{a^{3}-b^{3}}{a+b}+\frac{b^{3}-c^{3}}{b+c}+\frac{c^{3}-a^{3


  • Please log in to reply
Chủ đề này có 4 trả lời

#1
manhhung2013

manhhung2013

    Sĩ quan

  • Thành viên
  • 306 Bài viết

Cho a, b, c là các số thực dương. Chứng minh rằng:

$\left | \frac{a^{3}-b^{3}}{a+b}+\frac{b^{3}-c^{3}}{b+c}+\frac{c^{3}-a^{3}}{c+a} \right |\leqslant \frac{1}{4}\left [(a-b)^{2}+(b-c)^{2}+(c-a)^{2} \right ]$


đừng nghĩ LIKE và LOVE giống nhau...
giữa LIKE và LOVE chữ cái I đã chuyển thành O,tức là Important:quan trọng đã trở thành Only:duy nhất.
chữ cái K đã chuyển thành V:Keen:say mê đã trở thành Vascurla :ăn vào mạch máu.
vì thế đừng hỏi tại sao
lim(LIKE)=LOVE nhưng lim(LOVE) =

 


#2
Kamii0909

Kamii0909

    Trung sĩ

  • Thành viên
  • 157 Bài viết
Có vẻ như $\frac{1}{4}$ chưa phải hằng số tốt nhất.
Bình phương lên, điều phải chứng minh tương đương.
$$4\prod(a-b)^2 (ab+bc+ca)^2 \leq \prod (a+b)^2 (\sum a^2-bc)^2$$
Đổi về pqr.
$$ \dfrac{4q^2}{27} [4(p^2-3q)^3 -(2p^3-9pq+27r)^2] \leq (pq-r)^2(p^2-3q)^2 $$
$$L.H.S \leq \dfrac{16q^2(p^2-3q)^3}{27}$$
Ta quy điều phải chứng minh về
$$\dfrac{16q^2(p^2-3q)}{27} \leq (pq-r)^2$$
Có $$pq-r \geq \dfrac{8pq}{9}$$
Thay vào và biến đổi, bất đẳng thức tương đương với
$$q^2(\frac{p^2}{3} +3q) \geq 0$$
Hiển nhiên đúng.

#3
Nguyenhuyen_AG

Nguyenhuyen_AG

    Trung úy

  • Thành viên nổi bật 2016
  • 945 Bài viết

Có vẻ như $\frac{1}{4}$ chưa phải hằng số tốt nhất.
Bình phương lên, điều phải chứng minh tương đương.
$$4\prod(a-b)^2 (ab+bc+ca)^2 \leq \prod (a+b)^2 (\sum a^2-bc)^2$$

 

Bất đẳng thức này tương đương với

\[\prod (a+b) \sum (a^2-bc) \geqslant 2(ab+bc+ca)\prod(|a-b|).\]

Ta có

\[\prod (a+b) \sum (a^2-bc) - 2(ab+bc+ca)\prod(a-b) = \sum b(b+c)(c+a)(a-b)^2 \geqslant 0,\]

\[\prod (a+b) \sum (a^2-bc) + 2(ab+bc+ca)\prod(a-b) = \sum a(b+c)(c+a)(a-b)^2 \geqslant 0.\]

Từ đó suy ra điều phải chứng minh.


Bài viết đã được chỉnh sửa nội dung bởi Nguyenhuyen_AG: 29-01-2017 - 16:29

Nguyen Van Huyen
Ho Chi Minh City University Of Transport

#4
Kamii0909

Kamii0909

    Trung sĩ

  • Thành viên
  • 157 Bài viết

Cho a, b, c là các số thực dương. Chứng minh rằng:
$\left | \frac{a^{3}-b^{3}}{a+b}+\frac{b^{3}-c^{3}}{b+c}+\frac{c^{3}-a^{3}}{c+a} \right |\leqslant \frac{1}{4}\left [(a-b)^{2}+(b-c)^{2}+(c-a)^{2} \right ]$

Hằng số tốt nhất cho bất đẳng thức này khá xấu và có thể tìm bằng dồn biến toàn miền.
Cho $$a=0,b=2,c=1+ \sqrt{3}+\sqrt{2} \cdot 3^\frac{1}{4}$$ thì $k \geq \sqrt{\dfrac{2 \cdot \sqrt{3}-9}{9}}$

Bài viết đã được chỉnh sửa nội dung bởi Kamii0909: 06-02-2017 - 19:31


#5
Nguyenhuyen_AG

Nguyenhuyen_AG

    Trung úy

  • Thành viên nổi bật 2016
  • 945 Bài viết

Hằng số tốt nhất cho bất đẳng thức này khá xấu và có thể tìm bằng dồn biến toàn miền.
Cho $$a=0,b=2,c=1+ \sqrt{3}+\sqrt{2} \cdot 3^\frac{1}{4}$$ thì $k \geq \sqrt{\dfrac{2 \cdot \sqrt{3}-9}{9}}$

 

Đây là bài bất trong đề VMEO IV.


Nguyen Van Huyen
Ho Chi Minh City University Of Transport




0 người đang xem chủ đề

0 thành viên, 0 khách, 0 thành viên ẩn danh